LSAT and Law School Admissions Forum

Get expert LSAT preparation and law school admissions advice from PowerScore Test Preparation.

 SherryZ
  • Posts: 124
  • Joined: Oct 06, 2013
|
#11960
Hi there! Thank you in advance for helping me!!

June 2001 LSAT, Sec 3 LR, #26:

Although I chose the right answer (lucky ;) ), however, I found that B is very attractive, which cost me time to think about it :(

Could you explain why B is wrong?? ( BTW, I am bad at doing "Play Role" questions :( )

---Sherry
 Jacques Lamothe
PowerScore Staff
  • PowerScore Staff
  • Posts: 50
  • Joined: Sep 24, 2013
|
#11963
Hi Sherry,

This is definitely a tricky question. Answer choice (B) seems really attractive because you can easily imagine the information given in the stimulus being used as evidence that the media decides what to cover and the extent of its coverage based on public interest. The way to choose between (B) and (E) comes from the way in which the argument is presented.

The claim that the media decides what to cover depending on the public's interest is not presented as a conclusion that other information in the stimulus proves. Instead, it is introduced in the last sentence with the phrase "After all," suggesting that it is something the author assumes. With this in mind, the proposition that the public is now more interested in crime does not appear to be functioning as evidence for that claim.

You were right to pick (E) in the end, since the proposition that the pubic is currently more interested in crime appears in a sentence in which it is framed as a better explanation than higher crime rates for increased media coverage of crime.

I hope that this explanation helps! Thanks for posting your question!

Jacques
 jonwg5121
  • Posts: 38
  • Joined: Jun 06, 2015
|
#20239
Hi,

Can you please go over #26 and how to best approach this problem. Also, can you please explain why (A) is not correct. Thank you!
User avatar
 Dave Killoran
PowerScore Staff
  • PowerScore Staff
  • Posts: 5852
  • Joined: Mar 25, 2011
|
#20241
Hi Jon,

Thanks for the question. In reading this problem, the second sentence is the one that really jumps out at me. The first sentence sets up a circumstance: the media covers crime more no than they did 10 years ago. The second sentence then does two things: it first rejects one possible cause of the situation, and then it states what the author believes is the actual cause ("the public is now more interested in reading and hearing about crime"). The third sentence then follows up with an additional premise that explains why the cause is the cause.

So, from that reading, the whole problem is about causality and why one cause is the cause, and another possible cause isn't. You then get hit with a Method—Argument Part question, and you are questioned specifically about the role played by the second part of the second sentence. Well, this was stated as the real cause by the author, and it was offered up as an alternative to the possible cause of the crime rate increasing. So, of our answer choices (E) really matches well and is the correct answer.

Answer choice (A) has a few problems. First, is there a discussion of the appropriate amount of media coverage of crime? No, I don't see that anywhere in the argument, but (A) references devoting "more coverage to crime than the crime rate alone justifies" (italics added for emphasis). Second, the answer choice states that the coverage levels idea is the conclusion of the argument, but if it's not mentioned, then it's not going to be the conclusion, so that's wrong as well. Last, (A) states that the second part of the second sentence supports the conclusion, but really that second sentence as a whole is the conclusion of the argument itself. So, overall, there are problems all through that answer choice.

Please let me know if that helps. Thanks!
 EL16
  • Posts: 45
  • Joined: Jul 06, 2017
|
#38860
Hello,

I got this question correct, but came here just to double check my reasoning. When I read Dave's post above, I see that he mentioned that the 2nd sentence in the stimulus is the conclusion ("Yet this is not because..."). I guess I got lucky with this question, because I thought that the conclusion was actually the first sentence instead? The first sentence says that the media devotes more coverage to crime now than it did 10 years ago. I figured this is easily supported BY the 2nd sentence (and 3rd sentence as well), thereby making it the conclusion. My method to finding the first sentence as the conclusion was as follows:

Conclusion: The media covers more crime now than they did 10 years ago. Why? Because (premise 1) more people are interested in crime now, AND (premise 2) the media decides what to cover based on what people are interested in.

I am having trouble seeing how the 2nd sentence could be the conclusion instead of the first, as I am unsure of what the "why" factors are that would support that sentence if it is the conclusion. For example, the following explanation/reasoning just doesn't make sense:

Conclusion: The crime rate didn't increase, but people are just more interested in crime now. Why? Because the media devotes more coverage to crime.
^^This just doesn't seem logical to me. Any help would be appreciated!

Thanks,
Elana
 Adam Tyson
PowerScore Staff
  • PowerScore Staff
  • Posts: 5153
  • Joined: Apr 14, 2011
|
#39090
The first sentence is not a conclusion, Elana, because the author is not trying to prove it. There's no "therefore" implied in that claim - we are supposed to just accept it as established fact. The author is instead trying to prove that the cause of that established fact is not more crime, but more interest in crime. His evidence (the premises) can be found in the last sentence - the media covers what people are interested in. The structure looks like this:

Premise: The media covers what people are interested in

Premise: The media covers more crime than they used to

Conclusion: People must be more interested in crime than they used to be

While this argument could be rewritten slightly to make the first sentence the conclusion (the media covers what people are interested in and people are now more interested in crime, so the media must cover more crime), the correct interpretation has a lot to do with how the argument is presented. The author never tries to make a case for the media covering more crime, but presents it as a simple statement of fact. He isn't trying to prove "that" the media covers more crime, he is trying to demonstrate "why" they do so. The conclusion is not "therefore they do", but "therefore it's for this reason." See the difference?

Look for what it is that the author expects you to just accept without question, compared to what it is he is trying to convince you of. The former is a premise, the latter is a conclusion. Sometimes a somewhat "holistic" view of the argument in those terms can allow you to see things in a whole new (and better) light.

Keep up the good work!
 EL16
  • Posts: 45
  • Joined: Jul 06, 2017
|
#39098
Thank you Adam!

Get the most out of your LSAT Prep Plus subscription.

Analyze and track your performance with our Testing and Analytics Package.